Link Search Menu Expand Document

Soluciones de la lista de Ecuaciones

Te traemos las soluciones a los problemas de la Lista POLYNOMM del 28 de septiembre.

Álgebra

Teoría de números


Problema  1

Encuentra todos los pares de enteros positivos \((a,b)\) tales que \(a^2+b\) excede a \(b^2+a\) por 36.

Solución

En este problema, no es necesario realizar muchas operaciones:

\[\begin{align*} 36&=(a^2+b)-(b^2+a)\\[0.5em] &=a^2-b^2+b-a\\[0.5em] &=(a+b)(a-b)-(a-b)\\[0.5em] &=(a+b-1)(a-b) \end{align*}\]

Vemos que \(a+b+1>0\), por lo que \(a-b>0\) y ambos factores son los divisores positivos de 36. Notamos que \(a+b+1>a-b\), y ambos tienen distinta paridad (ya que su diferencia es \(2b+1\), un número impar), por lo que analizaremos los siguientes casos:

  1. \(a+b-1=36\), \(a-b=1\):

    Resolviendo el sistema de ecuaciones, se obtiene que \(a=19\) y \(b=18\).

  2. \(a+b-1=12\), \(a-b=3\):

    Nuevamente, al resolver el sistema de ecuaciones, \(a=8\) y \(b=5\).

  3. \(a+b-1=9\), \(a-b=4\):

    En este último caso, \(a=7\) y \(b=3\).

Por lo que todos los pares de enteros \((a,b)\) que satisfacen las condiciones son \((19,18)\), \((8,5)\) y \((7,3)\).

Problema  2

\(181^2\) puede escribirse como la diferencia de los cubos de dos enteros positivos. Encuentra la suma de dichos enteros.

Solución

Sea \(n\) el más pequeño de dichos enteros positivos. Entonces

\[181^2=(n+1)^3-n^3=3n^2+3n+1\]

Esto significa que:

\[\begin{align*} n(n+1)&=\dfrac{181^2-1}{3}\\[0.5em] &=\dfrac{180\cdot 182}{3}\\[0.5em] &=(2^2\cdot 3\cdot 5)(2\cdot 7\cdot 13)\\[0.5em] &=(2^3\cdot 13)\cdot(3\cdot 5\cdot 7)\\[0.5em] &=104\cdot 105 \end{align*}\]

La única solución positiva de esta ecuación es \(n=104\), por lo que la suma requerida es \(2n+1=209\).

Problema  3

Si \( y+4 = (x-2)^2 \), \(x+4 = (y-2)^2\) y \( x\ne y \), encuentra el valor de \( x^2+y^2\).

Solución

Podemos restar ambas ecuaciones y factorizar la diferencia de cuadrados:

\[\begin{align*} (y+4)-[x-4]&=(x-2)^2-[y-2]^2\\ y-x&=(x-2+y-2)(x-y)\\ 0&=(x-y)(x+y-4)+x-y\\ &=(x-y)(x+y-3) \end{align*}\]

Como \( x\ne y\), \( x-y\ne 0\), es decir, \( x+y=3 \) en la última ecuación para que se cumpla la igualdad. Ahora, sumamos las dos ecuaciones iniciales para terminar.

\[\begin{align*}\require{cancel} (y+4)+[x-4]&=(x-2)^2+[y-2]^2\\ x+y+\cancel{8}&=x^2+y^2-4(x+y)+\cancel{8}\\[0.5em] x^2+y^2&=5(x+y)\\ &=15 \end{align*}\]

Problema  4

Encuentra el valor de \( a_{}^{}x^5 + b_{}y^5\), si los números reales \( a \), \( b \), \( x \), y \( y\) satisfacen las siguientes ecuaciones:

\[\begin{align*} ax + by&=3\\ ax^2 + by^2&=7\\ ax^3 + by^3&=16\\ ax^4 + by^4&=42 \end{align*}\]

Solución

Sea \(S = (x + y)\) y \(P = xy\). Podemos usar la siguiente relación

\[(ax^n + by^n)(x + y) = (ax^{n + 1} + by^{n + 1}) + (xy)(ax^{n - 1} + by^{n - 1})\]

con las ecuaciones iniciales.

\[\begin{eqnarray*}(ax^2 + by^2)(x + y) & = & (ax^3 + by^3) + (xy)(ax + by) \\ (ax^3 + by^3)(x + y) & = & (ax^4 + by^4) + (xy)(ax^2 + by^2)\end{eqnarray*}\]

Lo anterior indica que

\[\begin{eqnarray*}7S & = & 16 + 3P \\ 16S & = & 42 + 7P\end{eqnarray*}\]

Resolviendo el sistema, obtenemos que \(S = - 14\) and \(P = - 38\). Podemos finalizar de la siguiente manera:

\[\begin{eqnarray*}(ax^4 + by^4)(x + y) & = & (ax^5 + by^5) + (xy)(ax^3 + by^3) \\ (42)(S) & = & (ax^5 + by^5) + (P)(16) \\ (42)( - 14) & = & (ax^5 + by^5) + ( - 38)(16)\end{eqnarray*}\]

Es decir:

\[ax^5 + by^5 = 20\]

Problema  5

Determina todas las soluciones reales \( x \), \( y \) y \( z \) del siguiente sistema de ecuaciones:

\[\begin{cases} x^3 - 3x = 4 - y \\ 2y^3 - 6y = 6 - z \\ 3z^3 - 9z = 8 - x\end{cases}\]

Solución

El siguiente sistema es equivalente al sistema inicial:

\[\left\{\begin{eqnarray*} x^3-3x-2&=&2-y\\2(y^3-3y-2)&=&2-z\\3(z^3-3z-2)&=&2-x\end{eqnarray*}\right.\]

Podemos factorizar cada ecuación de la siguiente manera

\[\left\{\begin{eqnarray*} (x+1)^2(x-2)&=&2-y\\2(y+1)^2(y-2)&=&2-z\\3(z+1)^2(z-2)&=&2-x\end{eqnarray*}\right.\]

Vemos que, si alguno de los números \( x \), \( y \) o \( z \) es igual a 2, todos son iguales a 2, pues el lado derecho de una ecuación sería un factor 0 en el lado izquierdo de otra ecuación.

Finalmente, si \( x, y, z \ne 2\), podemos multiplicar todas las ecuaciones, y obtener una contradicción, ya que los cuadrados de números reales no son negativos:

\[\begin{eqnarray*} x,y,z&\ne&2\\ \implies 6(x+1)^2(y+1)^2(z+1)^2(x-2)(y-2)(z-2)&=&(2-x)(2-y)(2-z)\\ \implies 6(x+1)^2(y+1)^2(z+1)^2&=&-1 \end{eqnarray*}\]

Por lo tanto, la única solución al sistema es \(x=y=z=2\).

Problema  6

Resuelva el siguiente sistema de ecuaciones

\[\begin{cases} x^2+2=x(y+z)\\ y^2+3=y(z+x)\\ z^2+4=z(x+y) \end{cases}\]

Solución

Primero, cada ecuación nos indica que \(x\), \( y \) y \( z \) son distintos de cero, pues en caso contrario, cada ecuación nos da una contradicción, como \( 2=0 \). Así, podemos realizar la división entre \( x \), \( y \) y \( z \). Lo anterior nos da como resultado el siguiente sistema de ecuaciones:

\[\begin{cases} x+\frac{2}{x}=y+z\\[0.5em] y+\frac{3}{y}=z+x\\[0.5em] z+\frac{4}{z}=x+y \end{cases}\]

Puedes sumar cada par de ecuaciones

\[\begin{align*} \left(\cancel{x}+\frac{2}{x}\right)+\left[\cancel{y}+\frac{3}{y}\right]&=(\cancel{y}+z)[z+\cancel{x}]\\ \frac{2}{x}+\frac{3}{y}&=2z\\ 2z&=\frac{3x+2y}{xy}\\ \left(y+\frac{3}{y}\right)+\left[z+\frac{4}{z}\right]&=(z+x)[x+y]\\ \frac{3}{y}+\frac{4}{z}&=2x\\ 2x&=\frac{4y+3z}{yz}\\ \left(z+\frac{4}{z}\right)+\left[x+\frac{2}{x}\right]&=(x+y)[y+z]\\ \frac{2}{x}+\frac{4}{z}&=2y\\ 2y&=\frac{4x+2z}{zx} \end{align*}\]
\[\implies 2xyz=3x+2y=4y+3z=4x+2z\]

Con lo anterior, podemos deducir las siguiente igualdadades:

\[\begin{align*} 4xyz-2xyz&=2xyz\\ 2(3x+2y)-(4y+3z)&=4x+2z\\ 2x&=5z\\ \implies x&=\frac{5z}{2}\wedge z=\frac{2x}{5}\\[0.5em] 2xyz&=2xyz\\ 4y+3z&=4x+2z\\ 4y+3z&=10z+12z\\ 4y&=9z\\ \implies y&=\frac{9z}{4}\\[0.5em] &=\frac{9x}{10} \end{align*}\]

Finalmente, podemos sustituir los valores de \( y \) y \( z \) en la primera ecuación del sistema inicial:

\[\begin{align*} x^2+2&=x(y+z)\\[0.5em] &=x\left(\frac{9x}{10}+\frac{2x}{5}\right)\\[0.5em] \implies 2&=\frac{3x^2}{10}\\[0.5em] \implies x^2&=\frac{20}{3}\\ \implies x_1=\frac{2\sqrt{15}}{3}&\wedge x_2=-\frac{2\sqrt{15}}{3}\\ \end{align*}\]

Con los valores posibles de \( x \) determinados, se concluye que las soluciones \( (x,y,z) \) del sistema son \( \left(\frac{2\sqrt{15}}{3}, \frac{3\sqrt{15}}{5}, \frac{4\sqrt{15}}{15}\right) \) y \( \left(-\frac{2\sqrt{15}}{3}, -\frac{3\sqrt{15}}{5}, -\frac{4\sqrt{15}}{15}\right) \).

Problema  7

Encuentra todas las 4-tuplas \( (a,b,c,d) \) de números reales que satisfacen el siguiente sistema de ecuaciones:

\[\begin{cases} (b+c+d)^{2010}=3a\\ (a+c+d)^{2010}=3b\\ (a+b+d)^{2010}=3c\\ (a+b+c)^{2010}=3d \end{cases}\]

Solución

Primero, dado que 2010 es un entero positivo par, \( a,b,c,d \ge 0 \). Dado que el sistema es equivalente en cualquier permutación de los 4 números, sin pérdida de generalidad podemos suponer que \( a\ge b\ge c\ge d\ge 0 \). En este caso:

\[\begin{eqnarray*} (b+c+d)^{2010}\ge (a+c+d)^{2010}&\ge&(a+b+d)^{2010}\ge(a+b+c)^{2010}\\ \implies (b+c+d)\ge(a+c+d)&\ge&(a+b+d)\ge(a+b+c)\\ \implies d\ge c&\ge&b\ge a \end{eqnarray*}\]

Esto quiere decir que \(a=b=c=d\). En este sentido, \((0,0,0,0)\) es claramente una solución, pero si todos los números son distintos de cero, basta con sustituir \( b \), \( c \) y \( d \) en la primera ecuación:

\[\begin{eqnarray*} (3a)^{2010}=3a&\implies&a=\frac{1}{3} \end{eqnarray*}\]

Por lo que la otra tupla solución es \(\left(\frac{1}{3},\frac{1}{3},\frac{1}{3},\frac{1}{3}\right)\).


Flotar

POLYNOMM. Regla y compás nunca ha de faltar...

Última modificación: 19 Oct 2020 (11:40 AM).